Question and Answers Forum

All Questions      Topic List

Relation and Functions Questions

Previous in All Question      Next in All Question      

Previous in Relation and Functions      Next in Relation and Functions      

Question Number 30173 by abdo imad last updated on 18/Feb/18

let u_n = Π_(k=1) ^n  (1+(k/n^2 ))  1. verify that x−(x^2 /2) ≤ln(1+x)≤x  2. prove that (u_n ) is convergente and find its limit.

$${let}\:{u}_{{n}} =\:\prod_{{k}=\mathrm{1}} ^{{n}} \:\left(\mathrm{1}+\frac{{k}}{{n}^{\mathrm{2}} }\right) \\ $$$$\mathrm{1}.\:{verify}\:{that}\:{x}−\frac{{x}^{\mathrm{2}} }{\mathrm{2}}\:\leqslant{ln}\left(\mathrm{1}+{x}\right)\leqslant{x} \\ $$$$\mathrm{2}.\:{prove}\:{that}\:\left({u}_{{n}} \right)\:{is}\:{convergente}\:{and}\:{find}\:{its}\:{limit}. \\ $$

Commented by abdo imad last updated on 21/Feb/18

let put ϕ(t)= ln(1+x)−(x−(x^2 /2))  for x≥0 we have  ϕ^′ (t)= (1/(1+x)) −(1−x)=((1−(1−x^2 ))/(1+x))= (x^2 /(1+x)) ≥0 so ϕ is  increasing  on [0,+∞[ we have ϕ(0)=0 ≥0 ⇒ ∀ x≥0  ϕ(x)≥0  we use the same method  for the function  ψ(x)= x−ln(1+x) and we show that ψ(x)≥0  2) we have ln(u_n )= Σ_(k=1) ^n ln(1+ (k/n^2 )) but   (k/n^2 ) −(k^2 /(2n^4 )) ≤ln (1+(k/n^2 )) ≤(k/n^2 ) ⇒  Σ_(k=1) ^n (k/n^2 )− Σ_(k=1) ^n   (k^2 /(2n^4 )) ≤ Σ_(k=1) ^n ln(1+(k/n^2 ))≤ Σ_(k=1) ^n  (k/n^2 ) ⇒  (1/(2n^2 ))n(n+1) −(1/(2n^4 ))((n(n+1)(2n+1))/6)≤ln(u_n )≤ (1/(2n^2 ))n(n+1)⇒  ((n+1)/(2n)) −(1/(12n^3 ))(n+1)(2n+1) ≤ln(u_n )≤ ((n+1)/(2n))  but  lim_(n→∞)  ((n+1)/(2n)) −(((n+1)(2n+1))/(12n^3 ))= (1/2) and lim_(n→∞) ((n+1)/(2n))=(1/2)  ⇒ lim_(n→∞) ln(u_n )=(1/2) ⇒ lim_(n→∞) u_n = (√e)    .

$${let}\:{put}\:\varphi\left({t}\right)=\:{ln}\left(\mathrm{1}+{x}\right)−\left({x}−\frac{{x}^{\mathrm{2}} }{\mathrm{2}}\right)\:\:{for}\:{x}\geqslant\mathrm{0}\:{we}\:{have} \\ $$$$\varphi^{'} \left({t}\right)=\:\frac{\mathrm{1}}{\mathrm{1}+{x}}\:−\left(\mathrm{1}−{x}\right)=\frac{\mathrm{1}−\left(\mathrm{1}−{x}^{\mathrm{2}} \right)}{\mathrm{1}+{x}}=\:\frac{{x}^{\mathrm{2}} }{\mathrm{1}+{x}}\:\geqslant\mathrm{0}\:{so}\:\varphi\:{is} \\ $$$${increasing}\:\:{on}\:\left[\mathrm{0},+\infty\left[\:{we}\:{have}\:\varphi\left(\mathrm{0}\right)=\mathrm{0}\:\geqslant\mathrm{0}\:\Rightarrow\:\forall\:{x}\geqslant\mathrm{0}\right.\right. \\ $$$$\varphi\left({x}\right)\geqslant\mathrm{0}\:\:{we}\:{use}\:{the}\:{same}\:{method}\:\:{for}\:{the}\:{function} \\ $$$$\psi\left({x}\right)=\:{x}−{ln}\left(\mathrm{1}+{x}\right)\:{and}\:{we}\:{show}\:{that}\:\psi\left({x}\right)\geqslant\mathrm{0} \\ $$$$\left.\mathrm{2}\right)\:{we}\:{have}\:{ln}\left({u}_{{n}} \right)=\:\sum_{{k}=\mathrm{1}} ^{{n}} {ln}\left(\mathrm{1}+\:\frac{{k}}{{n}^{\mathrm{2}} }\right)\:{but}\: \\ $$$$\frac{{k}}{{n}^{\mathrm{2}} }\:−\frac{{k}^{\mathrm{2}} }{\mathrm{2}{n}^{\mathrm{4}} }\:\leqslant{ln}\:\left(\mathrm{1}+\frac{{k}}{{n}^{\mathrm{2}} }\right)\:\leqslant\frac{{k}}{{n}^{\mathrm{2}} }\:\Rightarrow \\ $$$$\sum_{{k}=\mathrm{1}} ^{{n}} \frac{{k}}{{n}^{\mathrm{2}} }−\:\sum_{{k}=\mathrm{1}} ^{{n}} \:\:\frac{{k}^{\mathrm{2}} }{\mathrm{2}{n}^{\mathrm{4}} }\:\leqslant\:\sum_{{k}=\mathrm{1}} ^{{n}} {ln}\left(\mathrm{1}+\frac{{k}}{{n}^{\mathrm{2}} }\right)\leqslant\:\sum_{{k}=\mathrm{1}} ^{{n}} \:\frac{{k}}{{n}^{\mathrm{2}} }\:\Rightarrow \\ $$$$\frac{\mathrm{1}}{\mathrm{2}{n}^{\mathrm{2}} }{n}\left({n}+\mathrm{1}\right)\:−\frac{\mathrm{1}}{\mathrm{2}{n}^{\mathrm{4}} }\frac{{n}\left({n}+\mathrm{1}\right)\left(\mathrm{2}{n}+\mathrm{1}\right)}{\mathrm{6}}\leqslant{ln}\left({u}_{{n}} \right)\leqslant\:\frac{\mathrm{1}}{\mathrm{2}{n}^{\mathrm{2}} }{n}\left({n}+\mathrm{1}\right)\Rightarrow \\ $$$$\frac{{n}+\mathrm{1}}{\mathrm{2}{n}}\:−\frac{\mathrm{1}}{\mathrm{12}{n}^{\mathrm{3}} }\left({n}+\mathrm{1}\right)\left(\mathrm{2}{n}+\mathrm{1}\right)\:\leqslant{ln}\left({u}_{{n}} \right)\leqslant\:\frac{{n}+\mathrm{1}}{\mathrm{2}{n}}\:\:{but} \\ $$$${lim}_{{n}\rightarrow\infty} \:\frac{{n}+\mathrm{1}}{\mathrm{2}{n}}\:−\frac{\left({n}+\mathrm{1}\right)\left(\mathrm{2}{n}+\mathrm{1}\right)}{\mathrm{12}{n}^{\mathrm{3}} }=\:\frac{\mathrm{1}}{\mathrm{2}}\:{and}\:{lim}_{{n}\rightarrow\infty} \frac{{n}+\mathrm{1}}{\mathrm{2}{n}}=\frac{\mathrm{1}}{\mathrm{2}} \\ $$$$\Rightarrow\:{lim}_{{n}\rightarrow\infty} {ln}\left({u}_{{n}} \right)=\frac{\mathrm{1}}{\mathrm{2}}\:\Rightarrow\:{lim}_{{n}\rightarrow\infty} {u}_{{n}} =\:\sqrt{{e}}\:\:\:\:. \\ $$

Terms of Service

Privacy Policy

Contact: info@tinkutara.com